Paradox found in my first relativity sheet

In summary: The question was: If the rod shrinks to 0.8m in its rest-frame, does it still fit through the slit?In summary, the rod does not fit through the slit if you view it from the slit's frame, but it does fit if you view it from the rod's frame.
  • #1
Toby_phys
26
0
We have got some SR work to do, however we have only had 1 introductory lecture. I have a problem with one of the problems.

Question: A rod of 1m parrallel to the x-axis, travels at un-relativistic speeds at 45 degrees. It passes through a slit - also parallel to the x-axis - that is 1.1 meters wide.

Now assume the rod travels at 0.8c, does it fit through, look at it from both viewpoints, the rod and the slit?

In short, my answer was: I only need to look at the x velocity, y velocity will not cause the slit to shrink. X vlecocity = 0.8cos45 c

If you look at it from the slit's frame, the rod shrinks to about 0.8m and still fits. However if you look at it from the rod's frame, the slit shrinks to 0.9m, the rod doesn't fit.

How is this paradox resolved?
 
Physics news on Phys.org
  • #2
What happens to the "parallel to the x-axis" condition?
The y velocity is relevant.
 
  • #3
Toby_phys said:
We have got some SR work to do, however we have only had 1 introductory lecture. I have a problem with one of the problems.

Question: A rod of 1m parrallel to the x-axis, travels at un-relativistic speeds at 45 degrees. It passes through a slit - also parallel to the x-axis - that is 1.1 meters wide.

Now assume the rod travels at 0.8c, does it fit through, look at it from both viewpoints, the rod and the slit?

In short, my answer was: I only need to look at the x velocity, y velocity will not cause the slit to shrink. X vlecocity = 0.8cos45 c

If you look at it from the slit's frame, the rod shrinks to about 0.8m and still fits. However if you look at it from the rod's frame, the slit shrinks to 0.9m, the rod doesn't fit.

How is this paradox resolved?
No. in the rod's rest-frame the slit does NOT shrink; it is perpendicular to the direction of motion, so is not affected.
 
  • #4
Capture.png


Here is a diagram. How is the slit perpendicular to the motion
 
  • #5
Toby_phys said:
Capture.png


Here is a diagram. How is the slit perpendicular to the motion

Sorry; I mis-read the question.
 

What is a paradox?

A paradox is a statement or situation that seems to contradict itself, but upon further examination, may actually be true or have a logical explanation.

What is relativity?

Relativity is a theory in physics that explains the relationship between space and time, and how they are affected by gravity and motion.

What is the first relativity sheet?

The first relativity sheet is likely a reference to Albert Einstein's first paper on the theory of relativity, "On the Electrodynamics of Moving Bodies" published in 1905.

How does a paradox relate to the theory of relativity?

The theory of relativity has been known to produce several paradoxes, such as the twin paradox and the grandfather paradox, which challenge our understanding of time and space.

What are some examples of paradoxes found in the first relativity sheet?

One example is the twin paradox, where one twin travels through space at high speeds and experiences less time than the other twin who remains on Earth. This seemingly contradicts the idea that time is constant for all observers. Another example is the grandfather paradox, where a person travels back in time and prevents their own existence, leading to a contradiction.

Similar threads

  • Introductory Physics Homework Help
Replies
4
Views
1K
  • Introductory Physics Homework Help
Replies
2
Views
842
  • Introductory Physics Homework Help
Replies
7
Views
1K
  • Special and General Relativity
3
Replies
78
Views
4K
  • Special and General Relativity
Replies
21
Views
990
  • Introductory Physics Homework Help
Replies
1
Views
2K
  • Special and General Relativity
2
Replies
39
Views
3K
  • Introductory Physics Homework Help
Replies
6
Views
2K
  • Introductory Physics Homework Help
Replies
2
Views
1K
  • Special and General Relativity
Replies
16
Views
1K
Back
Top